4
$\begingroup$

I've asked two years ago a post on Mathematics Stack Exchange, were provided two excellent answers. I'm asking on MathOverflow in the hope that some professor can to expand/improve (if it is possible) these results answering my question. The post has the same title and identifier 3757149 on Mathematics Stack Exchange.

I don't know if the following diophantine equation (problem) is in the literature. We consider the diophantine equation $$x^{m-1}(x+1)=y^{n-1}(y+1)\tag{1}$$

over integers $x\geq 2$ and $y\geq 2$ with $x>y$, and over integers $m\geq 2$ and $n\geq 2$. These are four integral variables $x,y,m$ and $n$. The solutions that I know for the problem $(1)$ are two, the solution $(x,y;m,n)=(3,2;2,3)$ and $(98,21;2,3)$.

Question 1. Do you know if this problem is in the literature? Alternatively, if this problem isn't in the literature can you find more solutions? Many thanks.

If the equation or problem $(1)$ is in the literature please refer it answering this question as a reference request, and I try to search and read the statements for new solutions from the literature. In other case compute more solutions or add upto what uppers limits you got evidence that there aren't more solutions.

I would like to know what work can be done with the purpose to know if the problem $(1)$ have finitely many solutions $(x,y;m,n)$.

Question 2. Are there finitely many solutions $(x,y;m,n)$ of stated problem $(1)$? I mean what relevant reasonings or heuristics you can to deduce with the purpose to study if the problem have finitely many solutions. Many thanks

If this second question is in the literature, please refer the literature answering this question as a reference request, and I try to search and read the statements from the literature.

$\endgroup$
11
  • 1
    $\begingroup$ Why not $\ x^m\cdot(x+1)\ =\ y^n\cdot(y+1)\ $ over natural $\ x\ y\ m\ n\,\ \ (x>y)$ ? (natural means positive integers). $\endgroup$
    – Wlod AA
    Sep 9, 2022 at 19:08
  • 1
    $\begingroup$ There are no other solutions for $(m,n)=(2,3)$ and $(m,n)=(2,4)$. $\endgroup$
    – user178594
    Mar 18, 2023 at 11:21
  • 1
    $\begingroup$ The equation for $(m,n)=(2,5)$ is actually an hyperelliptic curve $x^2+x=y^5+y^4$. $\endgroup$
    – user178594
    Mar 18, 2023 at 11:34
  • 1
    $\begingroup$ Feel free to expand your answer below as you consider it with previous computations and information. Many thanks for your interest in my question @JovanRadenkovic $\endgroup$
    – user142929
    Mar 18, 2023 at 16:38
  • 1
    $\begingroup$ Can you write a PDF file "On the Diophantine equation $x^{m-1}(x+1)=y^{n-1}(y+1)$"? $\endgroup$
    – user178594
    Mar 24, 2023 at 21:14

4 Answers 4

10
$\begingroup$

You can make a lot of progress if you're willing to assume a deep conjecutre. The $N$-variable generalization of the $abc$-conjecture (https://en.wikipedia.org/wiki/N_conjecture) applied to your equation (with $N=4$) says that if $\gcd(x,y)=1$ and if no subsum of the $4$-term sum $$ x^n + x^{n-1} - y^m - y^{m-1} $$ vanishes, then $$ \max\{ x^n, y^m \} \le C_\epsilon\cdot (xy)^{3+\epsilon}. \quad(*) $$ Since you're assuming that $x>y\ge2$, this first implies that $$ x^n \le C_\epsilon\cdot x^{6+2\epsilon}, $$ so there are only finitely many solutions with $n\ge7$. If we assume that $n\le6$, then $$ x^6 \le x^n \le x^n + x^{n-1} = y^m + y^{m-1} \le 2y^m, $$ so $x\le (2y^m)^{1/6}$. Then $(*)$ gives $$ y^m \le C_\epsilon (xy)^{3+\epsilon} \le C_\epsilon ((2y^m)^{1/6})^{3+\epsilon})y^{3+\epsilon} \le C'_\epsilon y^{(m/2+3)(1+\epsilon)}. $$ This shows that there are only finitely many solutions with $m>m/2+3$, so only finitely many solutions with $m>6$.

So now you're reduced to a finite number of exponents, and since you've assumed that $x>y$, you need to handle exponents satisfying $2\le n<m\le 6$. Presumably for each choice of $(m,n)$, there are only finitely many solutions. In any case, since you want to avoid $x=y$, you're looking at integral points on the curve $$ \frac{x^n + x^{n-1} - y^m - y^{m-1}}{x-y}=0. $$ For each $(m,n)$, there will only be infinitely many integer solutions if the polynomials has a linear or quadratic factor.

Finally, it's possible that the cases where some subsum vanishes can be treated directly, but in any case, the usual $abc$-conjecture will handle the three term vanishing sums, and the two term vanishing sums are easy.

The assumptions in this analysis include the assumption that $\gcd(x,y)=1$. If you don't want that assumption, you can write $x=ZX$ and $y=ZY$ with $Z=\gcd(x,y)$, so your equation becomes $$ ZX^n + X^{n-1} = Z^{m-n+1} Y^m + Z^{m-n} Y^{m-1}. $$ Now you can try applying the $4$-variable $abc$ conjecture to this equation.

$\endgroup$
5
  • 1
    $\begingroup$ In my answer if we set $m=n$, we get many genus zero curves for $m=n<30$. $\endgroup$
    – joro
    Sep 10, 2022 at 6:58
  • $\begingroup$ Many thanks I'm familiarized with this kind of reasonings in the context of an application of abc conjecture for the asymptotic Fermat's Last Theorem; and Mason's theorem. When I can I study your excellent answer. $\endgroup$
    – user142929
    Sep 10, 2022 at 12:04
  • $\begingroup$ @joro I don't doubt that one gets lots of genus zero curves. For each of those, there are fairly easy criteria for whether there can be infinitely many integer points, based on the number of points "at infinity" on the completion of the affine curve. More precisely, if the completion of the desingularization has 3 or more points at infinity, then Siegel's theorem says finitely many integer points. If there are one or two points at infinity, then one can us local/global criteria. Of course, one has to deal with the genus 0 curves that appear on a case-by-case basis. $\endgroup$ Sep 10, 2022 at 13:18
  • $\begingroup$ Aren't rational points on the curves over number fields, resulting from the parametrization related to the N-conjecture? $\endgroup$
    – joro
    Sep 10, 2022 at 14:39
  • $\begingroup$ I suggest to solve the equation for positive integer pairs $(m,n)$, where $m$ and $n$ are fixed small positive integers, especially if either $m=2$ or $\gcd(m,n)\gt1$. $\endgroup$
    – user178594
    Mar 26, 2023 at 15:51
2
$\begingroup$

Paper by L.D.Lehmer, "On a problem by Stormer", pp.57-79, Illinois J.Math. (received July 25, 1962; that's all the info about the paper that I have + a reprint) is related and should be certainly useful. It contains complete (extensive) tables of integers $\ N\ $ such that the greatest prime divisor $\ p|N\!\!\cdot\!(N-1)\ $ is $\ p\le 41.\ $ This easily leads to a quick numerical method that would either discover all solutions with this kind of maximal prime $\ p\ $ or would show that there are no new solutions but the two mentioned by OP (for such maximal primes $\ \le 41$).

A step in this direction can be as follows:

Let's consider the OP's equation with a minor shift of $\ m\ n$:

$$ x^m\cdot(x+1)\ =\ y^n\cdot(y+1) \tag2 $$ where $$ x>y\qquad\text{and}\quad x\ m\ y\ n\,\in\mathbb N \tag3$$

The following function should be helpful, $$ Q(t)\ :=\ \frac12\cdot\rho(t\cdot(t+1)) $$ where $\ \rho(s)\ $is defined as the product of all prime divisors of $\ s;\ $ for instance $$ \rho(1) = 1\quad\qquad \rho(8)=2\quad\qquad \rho(30)=\rho(900)=30 $$ etc. Each solution of (2) implies a solution of equation

$$ Q(x)\ =\ Q(y) \tag4 $$

Thus an introductory and fundamental step toward solving (2) is

$$ Q(t)\ =\ s \tag5 $$

For the sake of (2) we need $\ s\ $ that allows more than one solution $\ t;\ $ there are relatively few of such cases, and when we have one then it is more often that not, that there is no respective solution of (2), a small bunch of elementary theorems can remove most of the negative cases.

For instance:

$$ Q(2)=Q(3)=Q(8)\qquad=\qquad \mathbf3 $$ leads to solution $\ 3\cdot4\ =\ 2^2\cdot3\ $ of (2). Then we have

$$ Q(5)=Q(9)=Q(15)=Q(24)=Q(80)\qquad=\qquad \mathbf{3\cdot5} $$ Let's look, say, at $\ Q(5)=Q(24).\ $ If possible, it'd be induced by equation (2) of the following form:

$$ 24^m\cdot25\ =\ 5^n\cdot 6 $$ This would force $\ n=2\ $ hence $\ 24^m\cdot25=5^2\cdot6,\ $ i.e. $\ 24^m=6\ $ -- a contradiction hence no solution in this case. Another way to see it is that right away we would have m=1 -- a contradiction again.

Or, one more case for now, let's look at $\ Q(9)=Q(80)\ $ hence at $$ 80^n\cdot 81\ =\ 9^m\cdot 10 $$ Again, you can see that $\ n=1\ $ and $\ m=2\ $ -- a contradiction.

In order to gain extra efficiency, one needs some additional theorems.

$\endgroup$
1
  • 1
    $\begingroup$ Many thanks for this excellent and very nice answer. I'm impressed, your answer and the answers of the other (MSE and MO) users could do, in my opinion, an interesting article in mathematics. $\endgroup$
    – user142929
    Sep 12, 2022 at 10:01
2
$\begingroup$

The equation

$$x^{m-1}(x-1)=y^{n-1}(y-1)$$

is given under the conditions $x\gt y\gt1$ and $2\le m\lt n$.

Partial answer:

The case $(m,n)=(2,3)$ is actually the rank $1$ elliptic curve. In fact, there are no further solutions for $(m,n)=(2,3)$, as proved by the following Magma program:

E:=EllipticCurve([-432,15120]);
Rank(E);
Generators(E);
TorsionSubgroup(E);
X:=IntegralPoints(E);
S:=[[(Abs(a[2])-108)/216, (a[1]-12)/36,2,3]: a in X | IsIntegral((a[1]-12)/36) and IsIntegral((Abs(a[2])-108)/216)] cat [[(-Abs(a[2])-108)/216, (a[1]-12)/36,2,3]: a in X | IsIntegral((a[1]-12)/36) and IsIntegral((Abs(a[2])-108)/216)];
S;
[(S[n,1]*(S[n,1]+1))/2: n in [1..#S/2]];

finds all such integral solutions. The only nontrivial positive integer solutions are $(x,y)=(3,2)$ and $(x,y)=(98,21)$. However, there are infinitely many rational solutions to this equation. Corollary: The only triangular numbers which are also pentagonal pyramidal are $$0, 1, 6, 4851$$. This implies that the OEIS sequence https://oeis.org/A226499 is not only finite but also complete.

Also, the only pronic numbers of the form $k^3+k^2$ for some integer $k$ are $0, 2, 12, 9702$, the doubled triangular numbers which are also pentagonal pyramidal as proved by the following Magma program:

[(S[n,1]*(S[n,1]+1)): n in [1..#S/2]];

Theorem: There are no nontrivial solutions for $(m,n)=(2,4)$.

Proof:

Suppose $$n=x^2+x=y^4+y^3$$ $(2.4)$.

Multiply this equation by $4$ and add $1$ to both sides:

$$(2x+1)^2=4y^4+4y^3+1$$.

Thus we get:

$$(2x+1)^2=4y^4+4y^3+y^2-y^2+1$$

$$(2x+1)^2=4y^4+4y^3+y^2-(y^2-1)$$

$$(2x+1)^2=(2y^2+y)^2-(y^2-1)$$.

If $2x+1=m$, then the equation can also be rewritten as

$$m^2=(2y^2+y)^2-(y^2-1)$$.

The inequalities

$$(2y^2+y-1)^2=4y^4+4y^3-3y^2-2y+1<4y^4+4y^3+1$$

for $-\frac{2}{3}<y$ or $y>0$,

and

$$(2y^2+y+1)^2=4y^4+4y^3+5y^2-2y+1>4y^4+4y^3+1$$

for $-\frac{2}{5}<y$ or $y>0$ imply that either $y=0$ or $y^2-1$=0. Since $y^2-1$=0 has no real solutions besides $y=1$ or $y=-1$, this gives that $y$ is in $\{0, 1, -1\}$, giving only the integral solutions

$$(m,y)\in\{(-1,\pm1),(0,\pm1),(1,\pm3)\}$$.

They lead only to the trivial solutions to $(2.4)$.

I also found the four non-integral rational solutions

$$(m,y)\in\{(-\frac{2}{3},\pm\frac{7}{9}),(-\frac{2}{5},\pm\frac{23}{25})\}$$, but they lead to the non-integral rational solutions to $(2.4)$.

In fact, the equation is efficiently solvable if either $m=2$ or $GCD(m,n)\gt1$.

$\endgroup$
4
  • 1
    $\begingroup$ What a nice answer! $\endgroup$
    – user142929
    Feb 16, 2023 at 15:07
  • 1
    $\begingroup$ Can someone extend this results for general $m,n$? $\endgroup$
    – user178594
    Mar 2, 2023 at 16:14
  • 1
    $\begingroup$ This equation needs to be studied in literature. $\endgroup$
    – user178594
    Mar 27, 2023 at 7:04
  • $\begingroup$ My intention was try to deduce an interesting equation from the definition of the Dedekind psi function, Wikipedia has an article dedicated to this multiplicative function (if I remember well taking prime powers, $x^m$ and $y^n$ as particular values for the equation $\psi(x^m)=\psi(y^n)$). $\endgroup$
    – user142929
    Mar 27, 2023 at 13:59
1
$\begingroup$

EDIT This answer doesn't satisfy OP's assumptions. Currently keeping it because of existence of infinitely many solutions over extensions of the integers and the relation with N-conjecture.


If you set $m=n$ you have infinitely many solutions $x=y$.

If $m=n=2$, the solutions are $x=y,x= -y-1$.

If $m=n=3$ we have the factorization $(-x + y) \cdot (x^2 + x*y + y^2 + x + y)$. Maybe for deep reasons, the quadratic factor doesn't have solutions over the integers, but we believe it has infinitely many solutions over $\mathbb{Z}[i]$.

For $m=n=4$ we have a cubic factor, which is expected to be of genus $1$, and according to sage it is genus zero, which might give integral points.

For $ m=n < 30$, the higher degree factor is genus zero, probably some algebraic geometer will explain it.

$\endgroup$
5
  • 1
    $\begingroup$ OT's assumption was $\ x>y.\ $ Thus $\ m<n\ $ hence your discussion is a total miss. Furthermore, even when, against the OP's assumption, $\ m=n,\ $ then the situation is totally trivial, $\ x=y.$ Somehow, your neglecting the assumption about all parameters being positive (even $\ \ge 2$). Thus, once again, this your second part is irrelevant to the problem. $\endgroup$
    – Wlod AA
    Sep 10, 2022 at 7:22
  • $\begingroup$ We're talking here about positive real numbers (natural numbers). Just look at the inequalities. $\endgroup$
    – Wlod AA
    Sep 10, 2022 at 7:45
  • 1
    $\begingroup$ @WlodAA Thanks, you are right. $\endgroup$
    – joro
    Sep 10, 2022 at 7:46
  • $\begingroup$ When I can I'm going to accept an answer for this post, after I read the contributions. Many thanks for your excellent answer, I think that this time I'm going to choose the other answer. $\endgroup$
    – user142929
    Sep 10, 2022 at 12:01
  • $\begingroup$ If $x\gt y\ge 2$, then obviously $m\lt n$. $\endgroup$
    – user178594
    Mar 25, 2023 at 13:35

Your Answer

By clicking “Post Your Answer”, you agree to our terms of service and acknowledge you have read our privacy policy.

Not the answer you're looking for? Browse other questions tagged or ask your own question.